Critical Reasoning Flashcards
Whose average is better
I have come across a question in GMATPrep.
Two navel cadets - Steve and Robert- received the same the evaluation in only one of four areas. Each evaluation was on a scaore of 1 to 10
From which of the following statements can one determine whose average score was higher on the evaluation?
- Robert graduated at the top of his class
2.Steve scored higher than Robert in two areas and neither cadet scorted below 5 in any area - Steve’s lowest score was less than or equal to Robert’s highest score
- Robert Received a score of 10 in three areas and Steve did not score higher than 8 in any area.
5.Robert’s highest score and Steve’s lowest score were in the same area.
The explanation given is The answer is C. Since they score the same in only one area, Steves lowest socre was equal to Robert’s highest score. Hence, in the other three areas Steve scored higher than Robert. Thus, Steves’s average score is higher than Robert’s.
I am unable to understand the explanation.
When they tell less then or equal to how can we assume that it is equal to.
If St’s lowest score is less than Robert’s highest score then assume St - 3, 4,5,6 and Robert 6,7,8,9. here St’s lowest score 3 is less than Rober’s highest score 9. Robert has higher average than Steve.
Good post? |
Ans is D. Probably the Official Answer got copied wrong before you got hold of it or something.
To put simply, you may have overlooked the stem. They have one eval in common. If Rob got three 10s and Steve got none higher than 8, then their fourth scores are each 8s or lower but the same in any case. So Robert clearly got higher marks using D. And may I say that navel cadets seem unseemlier than naval cadets…
Since I just got posting elsewhere tonight about the validity of Official Answer’s, let me just say that this is a very rare occurence, or perhaps someone from gmatclub.com slipped in a bogus Official Answer again.
Last edited by check.stone; 02-09-2007 at 03:58 PM. Reason: Backtracking…slightly
Reply Reply With Quote
Good post? |
from GMATprep2
if x an integer is the median of the 5 numbers
X,3,1,12,8 GREATER THEN THE average of the 5 numbers?
1 x>6
2 x is greater then the median of the 5 numbers
Good post? |
imo E
1. X> 6
Median is some number present in the series which divides the series into two groups - so by giving x as 2000 will increase the average but median remains the same - not suff
- X > median of 5 numbers
That means x is not median and hence median can be 3 or 8. Average of number is (x+24)/5 - not sufficient
Combining both average is > 6 - but we cannot say if this is grater or less than median
- Teenagers are often priced out of the labor market by the government-mandated minimum-wage level because employers cannot afford to pay that much for extra help. Therefore, if Congress institutes a subminimum wage, a new lower legal wage for teenagers, the teenage unemployment rate, which has been rising since 1960, will no longer increase.
Which of the following, if true, would most weaken the argument above?
(A) Since 1960 the teenage unemployment rate has risen when the minimum wage has risen.
(B) Since 1960 the teenage unemployment rate has risen even when the minimum wage remained constant.
(C) Employers often hire extra help during holiday and warm weather seasons.
(D) The teenage unemployment rate rose more quickly in the 1970’s than it did in the 1960’s.
(E) The teenage unemployment rate has occasionally declined in the years since 1960.
Correct answer B.
Whats wrong with answer [/spoiler]A?
IMO B
Premise: there is an direct relationship between teenager wages and minimum wages.
The thing to understand is that teenager wages are not dependant on minimum wages or by lowering minimum wages.
A) but we are trying to lower the minim wage not increase it. This would not weeken the argument
B) correct answer because it shows that even if minimum wages are kept constant, the teenager wages have been increasing. This shows that the teenager wages are affected by other factors than minimum wages and is not dependent on minimum wages.
C) argument shows seasonality of the wages and does not talk about the relationship between teenager wages and minimim wages.
D) same as C
E) same as E[/u]
It takes 4 weeks for a team of 5 professional Window Washers working regular full time hours to completely clean every window of the Empire State building. The building’s owner demands that all the windows always be clean.Yet even if the 5 washers work consistently throughout their regular work week,they will not be able to finish cleaning all the windows before some windows will need cleaning.
Which of the following statements as must be true on the basis of the statements above ?
A) If an empire state building window is to be kept clean, it must be cleaned by a professional cleaner.
B)The owner’s demand for proper cleaning of all the windows will never be fulfilled.
C) If a team of 5 window washers cleans all the empire state building’s windows in less than 4 weeks, some windows will not be properly cleaned.
D) In order to ensure that all of the Empire State Building’s windows are clean, the owner must have his window washers work overtime.
E)Some Empire State Building windows must be cleaned more frequently than once every four weeks if they are to be kept clean.
OA:E..Why not C? Is ‘some windows not be properly cleaned’ in option C playing a spoilsport?
E)Some Empire State Building windows must be cleaned more frequently than once every four weeks if they are to be kept clean.
Now let us go through the options one by one and check:
A– we do not know because it is not mentioned in the argument that only professional cleaners can clean the windows. Incorrect.
B– out of context and incorrect.
C– we are not concerned here about windows being “properly cleaned”. That is not the moot point. We are concerned about cleaning of windows. C digresses from the topic even if it may be true.
D– again, out of context and incorrect.
E– is very much consistent with what has been written in the passage and so must be true.
An important takeaway from this problem is that we are concerned with finding the correct answer, not the one that sounds true or right.
Often patients with ankle fractures that are stable, and thus do not require surgery, are given follow-up x-rays because their orthopedists are concerned about possibly having misjudged the stability of the fracture. When a number of follow-up x-rays were reviewed, however, all the fractures that had initially been judged stable were found to have
healed correctly. Therefore, it is a waste of money to order follow-up x-rays of ankle fracture initially judged stable.
Which of the following, if true, most strengthens the argument?
A. Doctors who are general practitioners rather than orthopedists are
less likely than orthopedists to judge the stability of an ankle fracture
correctly.
B. Many ankle injuries for which an initial x-ray is ordered are revealed
by the x-ray not to involve any fracture of the ankle.
C. X-rays of patients of many different orthopedists working in several
hospitals were reviewed.
D. The healing of ankle fractures that have been surgically repaired is
always checked by means of a follow-up x-ray.
E. Orthopedists routinely order follow-up x-rays for fractures of bone
other than ankle bones.
As @hardik has pointed out earlier the correct answer to this question is (C). This question is from Official Verbal review (Blue edition) and the OA is (C).
(B) is incorrect. The reason stated is “Naturally many ankle injuries do not involve fractures - x rays may sometimes be used to determine this - but the argument concerns only cases where there have been ankle fractures”.
Here we don’t know what is meant by “Many” and how many of the ankle injuries are ankle fractures. Moreover the apt reason for striking out this choice is - the argument only concerned with ankle fractures and not injuries”.
(C) is correct. It strengthens the x-ray data that examined making it sufficiently representative of cases of ankle fracture judged to be stable by orthopedists. The answer choice adds the information that the data for the conclusion comes from many orthopedists working in many hospitals. So we have greater assurance that the x-ray data is representative, and hence the argument is made much stronger.
Thanks
Prashant
Fish currently costs about the same at seafood stores throughout Eastville and its surrounding suburbs. Seafood stores buy fish from the same wholesalers and at the same prices, and other business expenses have also been about the same. But new tax breaks will substantially lower the cost of doing business within the city. Therefore, in the future, profit margins will be higher at seafood stores within the city than at suburban seafood stores.
For the purposes of evaluating the argument, it would be most useful to know whether.
(A)more fish wholesalers are located within the city than in the surrounding suburbs.
(B) Any people who currently own seafood stores in the suburbs surrounding Eastville will relocate their businesses nearer to the city
(C) The wholesale price of fish is likely to fall in the future
(D)Fish has always cost about the same at seafood stores throughout Eastville and its surrounding suburbs.
(E) Seafood stores within the city will in the future set prices that are lower than those at suburban seafood stores.
Fish costs same uniformly in Eastville and surrounding suburbs.
Same wholesalers, same prices, same business expenses.
New tax breaks will lower cost of doing business WITHIN city.,
CONCLUSION : Therefore, in future, profit margins will be higher within city than in suburbs.
Which of the following would be useful information to know in order to evaluate above conclusion?
A. More or less, their profit margins can still be different just the same - not useful.
B. Nearer the city. So what? Is that within the city? And even if it is, do we know why they moved and how it is related to this issue of the profit margin? No.
C. If wholesale price of fish falls, rises or stays equal, it will still affect all these stores both inside and outside city, and will thus uniformly affect profit margin. This won’t help us validate that profit margin within city is more or not.
D. If it has always cost the same, so what:? Does that explain the possibilities in the future?
E. Ahah. If the ones in the city set lower prices than the suburban ones, then the amount that they save with the tax breaks could be counterbalanced by reduced revenue and make profit margins within and outside city limits equal or even possibly more in the suburban ones. Useful info.
Pick E.
- Which of the following, if true, provides evidence that most logically completes the argument below?
According to a widely held economic hypothesis, imposing strict environmental regulations reduces economic growth. This hypothesis is undermined by the fact that the states with the strictest environmental regulations also have the highest economic growth. This fact does not show that environmental regulations promote growth, however, since ______.
A. those states with the strictest environmental regulations invest the most in education and job training
B. Even those states that have only moderately strict environmental regulations have higher growth than those with the least-strict regulations
C. many states that are experiencing reduced economic growth are considering weakening their environmental regulations
D. after introducing stricter environmental regulations, many states experienced increased economic growth
E. even those states with very weak environmental regulations have experienced at least some growth
Answer A is correct because it states that economic growth is not connected to the level of environmental regulations. This answer provides an alternative explanation of this situation. Economic growth is directly related to the amount of funds invested in educations and job training. Therefore, economic growth is NOT the result of strict environmental regulations.
Answer E, on the other hand, states that countries with weak regulations also experience some growth. This answer is too broad. It does not give any evidence to support the stand that economic growth and environmental regulations are two independent factors.
_________________
Kasia
Senior Instructor
Master GMAT - the #1 rated GMAT course
• If you found my post helpful, please click the “thank” button and/or follow me.
• Take a 7 day free trial and find out why Master GMAT is the highest rated GMAT course - http://mastergmat.com
• Read Master GMAT reviews - http://reviews.beatthegmat.com/master-gmat
Does this CR needs technical knowledge, such as peat. flora and fauna? What is peat?
Opponents of peat harvesting in this country argue that it would alter the ecological balance of our peat-rich wetlands and that, as a direct consequence of this, much of the country’s water supply would be threatened with contamination. But this cannot be true, for in Ireland, where peat has been harvested for centuries, the water supply is not contaminated. We can safely proceed with the harvesting of peat.
Which one of the following, if true, most strengthens the argument?
(A) Over hundreds of years, the ecological balance of all areas changes slowly but significantly, sometimes to the advantage of certain flora and fauna.
(B) The original ecology of the peat-harvesting areas of Ireland was virtually identical to that of the undisturbed wetlands of this country.
(C) The activities of the other industries in coming years are likely to have adverse effects on the water supply of this country.
(D) The peat resources of this country are far larger than those of some countries that successfully harvest peat.
(E) The peat-harvesting industry of Ireland has been able to supply most of that country’s fuel for generations.
No OA
IMO B
_________________
Sudhanshu
(have lot of things to learn from all of you)
I also think that the OA is B.
I don’t think that you need technical or outside knowledge. The GMAT, however, does want you to FEEL confused
So you could substitute the word peat with anything. So you could say “Opponents of apple harvesting in this country…” It will all still work because the GMAT cannot require you to know outside things about peat. In some ways it can be a blessing to not know a lot about something in an argument because you may lean toward “trick” answer choices that seem to play to your knowledge.
Try reading and analyzing the passage again with the word apples and see if that helps.
Last edited by barcebal on Thu Aug 05, 2010 12:48 am; edited 1 time in total
OG-12 CR Q-91 Environmentalist
Wed May 02, 2012 11:31 pmThankQuoteEdit Tags
Elapsed Time: 00:00startlapstop
Environmentalist: The commissioner of the Fish and Game Authority would have the public believe that increases in the number of marine fish caught demonstrate that this resource is no longer endangered. This is a specious argument, as unsound as it would be to assert that the ever-increasing rate at which rain forests are being cut down demonstrates a lack of danger to that resource. The real cause of the increased fish-catch is a greater efficiency in using technologies that deplete resources.
The environmentalist’s statements, if true, best support which of the following as a conclusion?
(A) The use of technology is the reason for the increasing encroachment of people on nature.
(B) It is possible to determine how many fish are in the sea in some way other than by catching fish.
(C) The proportion of marine fish that are caught is as high as the proportion of rain forest trees that are cut
down each year.
(D) Modern technologies waste resources by catching inedible fish.
(E) Marine fish continue to be an endangered resource.
What I understand from the argument is:
The commissioner wants the public to believe that marine fish is not endangered anymore, whereas , it is the reverse case.
I think even A support’s the conclusion well enough by saying that technology is the root cause for encroachment ?
Please help
Thanks
Flag
transfer9858 wrote:
Whats the official answer?
I would say E because the environmentalist is saying, “ no, the fish are still not safe and your argument makes no sense because your just getting better and catching more fish. Also, if I were to get create a better technology to cut down trees, does that mean there are more trees and so trees aren’t in danger anymore? “ So at the end, fish are still endangered. Make sense?
Yes, the OA is E .
Thanks mate ! I get that
CR PROBLEM - Q1
Tue Aug 14, 2012 7:03 amThankQuoteEdit Tags
Elapsed Time: 00:00startlapstop
Policy Adviser: Freedom of speech is not only a basic human right, it is also the only rational policy for this government to adopt. When ideas are openly aired good idea flourish, silly proposals are easily recognized as such, and dangerous ideas can be responded to by forcing citizens to disseminate their thoughts in secret.
Which one of the following, if true, world most strengthen the argument?
(A) Most citizens would tolerate some limits on freedom of speech
(B) With or without a policy of freedom of speech, governments respond to dangerous ideas irrationally
(C) Freedom of religion and freedom of assembly are also basic human rights than governments must recognize
(D) Governments are less likely to be overthrown if they openly adopt a policy allowing freedom of speech
(E) Great ideas have flourished in societies that repress free speech as often as in those that permit it
OA-D
Policy Adviser: Freedom of speech is not only a basic human right, it is also the only rational policy for this government to adopt. When ideas are openly aired good idea flourish, silly proposals are easily recognized as such, and dangerous ideas can be responded to by forcing citizens to disseminate their thoughts in secret.
Which one of the following, if true, world most strengthen the argument?
Conclusion: Government should adopt Freedom of speech (FOS) policy.
Premise:
1. FOS is basic human right as well as only rational policy.
Answer should be something that tells obligation to have FOS policy. i.e. Govt will run effectively only in presence of this policy. Or it will not work in absence of FOS policy.
(A) Most citizens would tolerate some limits on freedom of speech -> It weakens saying, govt can do without FOS policy.
(B) With or without a policy of freedom of speech, governments respond to dangerous ideas irrationally. -> Somewhat towards conclusion, but it neighter strenghtens nor weakens. It only says Govt will act irrationally in absence of the policy.
(C) Freedom of religion and freedom of assembly are also basic human rights than governments must recognize. -> No tie with the conclusion.
(D) Governments are less likely to be overthrown if they openly adopt a policy allowing freedom of speech. -> It says Govt. will not be overthrown if FOS policy adopted. In other words, Govt. will be overthrown if FOS policy is not adopted. So, It is the answer.
(E) Great ideas have flourished in societies that repress free speech as often as in those that permit it. -> It repeats whatever said in argument and No tie with conclusion.
CR PROBLEM - Q2
Tue Aug 14, 2012 7:05 amThankQuoteEdit Tags
Elapsed Time: 00:00startlapstop
Naturalist: For decades we have known that the tuatara, a New Zealand reptile, has been approaching extinction on the South Island but since South Island tuatara were thought to be of the same species as North Island tuatara there was no need to protect them. New research indicates that the South Island tuatara are a distinct species, found only in that location. Because it is now known that, if the South Island tuatara are lost, an entire species will thereby be lost, human beings are now obliged to prevent their extinction, even if it means killing many of their unendangered natural predators.
Which one of the following principles most helps to justify the naturalists’ argumentation?
(A) In order to maximize the number of living things on Earth. Steps should be taken to preserve all local populations of animals.
(B) When an animal is in danger of dying, there is an obligation to help save its life, if doing so would not interfere with the health or well-being of other animals or people.
(C) The threat of local extinction imposes no obligation to try to prevent that extinction, whereas the threat of global extinction does impose such an obligation.
(D) Human activities that either intentionally or unintentionally threaten the survival of an animal species ought to be curtailed.
(E) Species that are found in only one circumscribed geographical region ought to be given more care and attention than are other species because they are more vulnerable to extinction.
Confused between C and E. OA - C
confuse mind wrote:
Naturalist: For decades we have known that the tuatara, a New Zealand reptile, has been approaching extinction on the South Island but since South Island tuatara were thought to be of the same species as North Island tuatara there was no need to protect them. New research indicates that the South Island tuatara are a distinct species, found only in that location. Because it is now known that, if the South Island tuatara are lost, an entire species will thereby be lost, human beings are now obliged to prevent their extinction, even if it means killing many of their unendangered natural predators.
Which one of the following principles most helps to justify the naturalists’ argumentation?
(A) In order to maximize the number of living things on Earth. Steps should be taken to preserve all local populations of animals.
(B) When an animal is in danger of dying, there is an obligation to help save its life, if doing so would not interfere with the health or well-being of other animals or people.
(C) The threat of local extinction imposes no obligation to try to prevent that extinction, whereas the threat of global extinction does impose such an obligation.
(D) Human activities that either intentionally or unintentionally threaten the survival of an animal species ought to be curtailed.
(E) Species that are found in only one circumscribed geographical region ought to be given more care and attention than are other species because they are more vulnerable to extinction.
The answer is C for the following reason: Before, when the people thought the North tuatara was the same as the South one, no one wanted to protect it. Now, they know they are different and want to protect it. They will even sacrifice other animals in order to protect the species. Statement C says “The threat of local extinction imposes no obligation to try to prevent that extinction, whereas the threat of global extinction does impose such an obligation.” That means that they are not willing to protect a species when it is will only locally die out (when there are other places with the species, {the NORTH}), but when they might be extincted globally (north and south are different), they will act to protect it.
Statement E is irrelevant to the issue at hand. E states that Species only found in one area should be given more care then other species. The argument is not really dealing with this issue. Sure, the species is only in one region, but the point is that it is going to be extinct. If it lived in a large area or a small area is irrelevant.
Hope this helps.
Fresh potatoes generally cost about $2 for a 10-pound bag, whereas dehydrated instant potatoes cost, on average, about $3 per pound. It can be concluded that some consumers will pay 15 times as much for convenience, since sales of this convenience food continue to rise.
Which of the following, if true, indicates that there is a major flaw in the argument above?
(A) Fresh potatoes bought in convenient 2-pound bags are about $1 a bag, or 2 1/2 times more expensive than fresh potatoes bought in 10-pound bags.
(B) Since fresh potatoes are 80 percent water, one pound of dehydrated potatoes is the equivalent of 5 pounds of fresh potatoes.
(C) Peeled potatoes in cans are also more expensive than the less convenient fresh potatoes.
(D) Retail prices of dehydrated potatoes have declined by 20 percent since 1960 to the current level of about $3 a pound.
(E) As a consequence of labor and processing costs, all convenience foods cost more than the basic foods from which they are derived.
gmat009 wrote:
Fresh potatoes generally cost about $2 for a 10-pound bag, whereas dehydrated instant potatoes cost, on average, about $3 per pound. It can be concluded that some consumers will pay 15 times as much for convenience, since sales of this convenience food continue to rise.
Which of the following, if true, indicates that there is a major flaw in the argument above?
(A) Fresh potatoes bought in convenient 2-pound bags are about $1 a bag, or 2 1/2 times more expensive than fresh potatoes bought in 10-pound bags.
(B) Since fresh potatoes are 80 percent water, one pound of dehydrated potatoes is the equivalent of 5 pounds of fresh potatoes.
(C) Peeled potatoes in cans are also more expensive than the less convenient fresh potatoes.
(D) Retail prices of dehydrated potatoes have declined by 20 percent since 1960 to the current level of about $3 a pound.
(E) As a consequence of labor and processing costs, all convenience foods cost more than the basic foods from which they are derived.
According to PowerScore CR Bible, this question belongs to family 2. i.e. we need to find the flaw in the conclusion by considering each answer choice as 100% correct.
If we look at the choice (B), it states that
1 Pound of Dehydrated potatoes = 5 pound of fresh potatoes.
That means, 2 pound of dehydrated potatoes = 10 pound of fresh potatoes. It means consumer will pay actually paying 6$ for the equivalent of 10 pounds of fresh potatoes. So, rate is increased by 3 times instead of 15 times.
Hence choose (B)
Option (C) is out of scope, because it talks about Peeled potatoes, which is out of scope in this context.
Hope this helps…
There are about 75 brands of microwave popcorn on the market; altogether, they account for a little over half of the money from sales of microwave food products. It takes three minutes to pop corn in the microwave, compared to seven minutes to pop corn conventionally. Yet by weight, microwave popcorn typically costs over five times as much as conventional popcorn. Judging by the popularity of microwave popcorn, many people are willing to pay a high price for just a little additional convenience.
If the statements in the passage are true, which one of the following must also be true?
(A) No single brand of microwave popcorn accounts for a large share of microwave food product sales.
(B) There are more brands of microwave popcorn on the market than there are of any other microwave food product.
(C) By volume, more microwave popcorn is sold than is conventional popcorn.
(D) More money is spent on microwave food products that take three minutes or less to cook than on microwave food products that take longer to cook.
(E) Of the total number of microwave food products on the market, most are microwave popcorn products.
confuse mind wrote:
There are about 75 brands of microwave popcorn on the market; altogether, they account for a little over half of the money from sales of microwave food products. It takes three minutes to pop corn in the microwave, compared to seven minutes to pop corn conventionally. Yet by weight, microwave popcorn typically costs over five times as much as conventional popcorn. Judging by the popularity of microwave popcorn, many people are willing to pay a high price for just a little additional convenience.t know how many products there are.
Economist: During a recession, a company can cut personnel costs either by laying off some employees without reducing the wages of remaining employees or by reducing the wages of all employees without laying off anyone. Both damage morale, but layoffs damage it less, since the aggrieved have, after all, left. Thus, when companies must reduce personnel costs during recessions, they are likely to lay off employees.
Which one of the following, if true, most strengthens the economist’s reasoning?
(A) Employee morale is usually the primary concern driving companies’ decisions about whether to lay off employees or to reduce their wages.
(B) In general, companies increase wages only when they are unable to find enough qualified employees.
(C) Some companies will be unable to make a profit during recessions no matter how much they reduce personnel costs.
(D) When companies cut personnel costs during recessions by reducing wages, some employees usually resign.
(E) Some companies that have laid off employees during recessions have had difficulty finding enough qualified employees once economic growth resumed.
OA after some Reply..
LSAT Set 56 CR
Economist: During a recession, a company can cut personnel costs either by laying off some employees without reducing the wages of remaining employees or by reducing the wages of all employees without laying off anyone.
[Argument: Both damage morale, but layoffs damage it less, since the aggrieved have, after all, left.
Result: Thus, when companies must reduce personnel costs during recessions, they are likely to lay off employees.
We need to find the link between the argument and the result.
Which one of the following, if true, most strengthens the economist’s reasoning?
(A) Employee morale is usually the primary concern driving companies’ decisions about whether to lay off employees or to reduce their wages.
Correct answer. Links the argument and result and strengthens the argument. If morale is the key concern and laying off has lesser impact on morale then laying off is the best option.
(B) In general, companies increase wages only when they are unable to find enough qualified employees.
Incorrect and out of context.
(C) Some companies will be unable to make a profit during recessions no matter how much they reduce personnel costs.
Incorrect and out of context.
(D) When companies cut personnel costs during recessions by reducing wages, some employees usually resign.
Incorrect. Provides further information on the impact of cutting wages but does not link the result and argument.
(E) Some companies that have laid off employees during recessions have had difficulty finding enough qualified employees once economic growth resumed.
Incorrect and out of context
OA after some Reply..
Vitcorp, a manufacturer, wishes to make its information booth at an industry convention more productive in terms of boosting sales. The both offers information introducing the company’s new products and services. To achieve the desired result, Vitacorp’s marketing department will attempt to attract more people to the booth. The marketing director’s first measure was to instruct each salesperson to call his or her five best customers and personally invite them to visit the booth.
Which of the following, if true, most strongly supports the prediction that the marketing director’s first measure will contribute to meeting the goal of boosting sales?
(A) Vitacorp’s salespeople routinely inform each important customer about new products and services as soon as the decision to launch them has been made.
(B) Many of Vitacorp’s competitors have made plans for making their own information booths more productive in increasing sales.
(C) An infomation booth that is well attended tends to attract visitors who would not otherwise have attended the booth.
(D) Most of Vitacorp’s best customers also have business dealings with Vitcorp’s competitors.
(E) Vitacorp has fewer new products and services available this year than it had in previous years.
This is an OG question - Q82 in Verbal Review. I am not convinced with OG’s answer. Its (C). I think (C) is too general in this context.
Conc: to attract more people to the booth and boost sales
(A) Vitacorp’s salespeople routinely inform each important customer about new products and services as soon as the decision to launch them has been made. (informing doesnt mean more ppl will be at the booth)
(B) Many of Vitacorp’s competitors have made plans for making their own information booths more productive in increasing sales. (we are more concerned abt Vitacorp n its sales. OOS)
(C) An infomation booth that is well attended tends to attract visitors who would not otherwise have attended the booth. (in line with the Director’s strategy. More crowd, more ppl tend to visit. correct)
(D) Most of Vitacorp’s best customers also have business dealings with Vitcorp’s competitors. (not pertinent to discussion. OOS)
(E) Vitacorp has fewer new products and services available this year than it had in previous years. (again OOS)
Hence (C)
A recent study of people who had successfully lost weight and implemented regular exercise routines in their twenties found that, by the age of forty, most had gained back the weight, stopped exercising regularly, or both. Surprisingly, among the study’s subjects who had hired personal trainers to help them lose weight and exercise in their twenties, an even higher percentage had regained the weight or stopped exercising than among subjects who had not hired such trainers. The researchers concluded not only that remaining on an effective weight management and exercise program is difficult, but also that personal trainers are largely ineffective in helping people to remain on an effective weight management and exercise program.
Which of the following, if true, most weakens the argument in the passage above?
1) Subjects in the study who had hired personal trainers had, on average, more free time to exercise than did those who had not hired such trainers.
2) Some people hire personal trainers for sport-specific training or to rehabilitate injuries, rather than to lose weight or implement a regular exercise program.
3) The average person’s metabolism slows significantly between the ages of thirty and forty, making it more difficult for people aged forty or older to avoid weight gain.
4) Many of the personal trainers hired by the people in the study were also dietitians who helped their clients design meal plans.
5) Most people who hire personal trainers do so, at least in part, because they lack sufficient motivation to remain on a diet or exercise regimen by themselves.
E
livery wrote:
A recent study of people who had successfully lost weight and implemented regular exercise routines in their twenties found that, by the age of forty, most had gained back the weight, stopped exercising regularly, or both. Surprisingly, among the study’s subjects who had hired personal trainers to help them lose weight and exercise in their twenties, an even higher percentage had regained the weight or stopped exercising than among subjects who had not hired such trainers. The researchers concluded not only that remaining on an effective weight management and exercise program is difficult, but also that personal trainers are largely ineffective in helping people to remain on an effective weight management and exercise program.
Which of the following, if true, most weakens the argument in the passage above?
1) Subjects in the study who had hired personal trainers had, on average, more free time to exercise than did those who had not hired such trainers. Having more free time doesn’t make any difference. We have no information what are they doing in their free time.
2) Some people hire personal trainers for sport-specific training or to rehabilitate injuries, rather than to lose weight or implement a regular exercise program.We are concerned only about those trainers hired to help the people lose weight as the author exactly specifies in the paragraph.
3) The average person’s metabolism slows significantly between the ages of thirty and forty, making it more difficult for people aged forty or older to avoid weight gain. This has nothing to do with trainers and it affects equally the people with or without trainer.
4) Many of the personal trainers hired by the people in the study were also dietitians who helped their clients design meal plans. We know that the people hired personal trainers to lose weight and exercise, so dieting and designing meal plans might have been part of the lose weight program. This option doesn’t provide any new information about how efficient were those trainers.
5) Most people who hire personal trainers do so, at least in part, because they lack sufficient motivation to remain on a diet or exercise regimen by themselves.We are learning that those people who hire personal trainers have less motivation to diet and exercise by themselves than the other people who do not hire trainers, so the problem is not with the trainers but with the trainees . If the trainers are not responsible for the inefficiency of the program the argument has no basis.
The argument of the paragraph is that because the percentage of people who regain weight is higher among those who hired trainer than among those who didn’t hire trainer the author concludes that trainers are largely ineffective. So we are looking for the option that weakens this argument.
IMO option 5) is the right answer.
The Maxilux car company’s design for its new luxury model, the Max 100, included a special design for the tires that was intended to complement the model’s image. The winning bid for supplying these tires was submitted by Rubco. Analysts concluded that the bid would only just cover Rubco’s costs on the tires, but Rubco executives claim that winning the bid will actually make a profit for the company.
Which of the following, if true, most strongly justifies the claim made by Rubco’s executives?
(A) In any Maxilux model, the spare tire is exactly the same make and model as the tires that are mounted on the wheels.
(B) Rubco holds exclusive contracts to supply Maxilux with the tires for a number of other models made by Maxilux.
(C) The production facilities for the Max 100 and those for the tires to be supplied by Rubco are located very near each other.
(D) When people who have purchased a carefully designed luxury automobile need to replace a worn part of it, they almost invariably replace it with a part of exactly the same make and type.
(E) When Maxilux awarded the tire contract to Rubco, the only criterion on which Rubco’s bid was clearly ahead of its competitors’ bids was price.
D
wrote:
The Maxilux car company’s design for its new luxury model, the Max 100, included a special design for the tires that was intended to complement the model’s image. The winning bid for supplying these tires was submitted by Rubco. Analysts concluded that the bid would only just cover Rubco’s costs on the tires, but Rubco executives claim that winning the bid will actually make a profit for the company.
Which of the following, if true, most strongly justifies the claim made by Rubco’s executives?
(A) In any Maxilux model, the spare tire is exactly the same make and model as the tires that are mounted on the wheels.
(B) Rubco holds exclusive contracts to supply Maxilux with the tires for a number of other models made by Maxilux.
(C) The production facilities for the Max 100 and those for the tires to be supplied by Rubco are located very near each other.
(D) When people who have purchased a carefully designed luxury automobile need to replace a worn part of it, they almost invariably replace it with a part of exactly the same make and type.
(E) When Maxilux awarded the tire contract to Rubco, the only criterion on which Rubco’s bid was clearly ahead of its competitors’ bids was price.
1)Can anyone please explain me why we are eliminating B,C and D options?
Option B)As per my understanding we are eliminating B as it talks about the other models of Maxilux company but our concern is how Rubco makes profit on the winning bid for Max 100 ……..Please let me know if my understanding is correct on this answer?
Option C talks about less production cost so it should help Rubco in earning profits. Why are we eliminating this option?
Please explain why D is correct?
2)One more question is that in the passage it is mentioned that the Analyst concluded that the bid would only just cover Rubco’s cost on tires.Is this a premise/fact ?Do we have to consider this as a fact? Can we attack what analyst concluded?
Premise: The bid to supply tires for the Max 100 will just cover Rubco’s costs.
Conclusion: As a result of winning the bid, Rubco will make a profit.
For the conclusion to be valid, what must be true?
As a result of winning the bid, Rubco must be able to make a profit in SOME OTHER WAY.
Answer choice D: When people who have purchased a carefully designed luxury automobile need to replace a worn part of it, they almost invariably replace it with a part of exactly the same make and type.
This answer choice shows how Rubco will make a profit: when purchasers of the Max 100 need a new tire, they will buy the specially-designed tire made by Rubco.
The correct answer is D.
Reasons to eliminate:
A: The spare tire is irrelevant. It is given as a PREMISE – as a FACT not in dispute – that supplying tires for the Max 100 will not yield a profit.
B: Since these contracts are already held by Rubco, they are not affected by the bid to supply tires for the Max 100.
C: How the tires are produced is irrelevant. It is given as a PREMISE – as a FACT not in dispute – that supplying tires for the Max 100 will not yield a profit.
E: Irrelevant. It is given as a PREMISE – as a FACT not in dispute – that Rubco supplied the winning bid. The correct answer must show how – as a RESULT of winning the bid – Rubco will realize a profit.
- Many physicists claim that quantum mechanics may
ultimately be able to explain all fundamental phenomena,
and that, therefore, physical theory will soon be complete.
However, every theory in the history of physics that was
thought to be final eventually had to be rejected for failure to
explain some new observation. For this reason, we can
expect that quantum mechanics will not be the final theory.
Which one of the following arguments is most similar in
its reasoning to the argument above?
(A) Only a few species of plants now grow in very dry climates;
therefore, few species of animals can live in those climates.
(B) Four companies have marketed a new food processing
product; therefore, a fifth company will not be able to market a
similar product.
(C) Your sister is a very good chess player but she has
never won a chess tournament; therefore, she will not
win this chess tournament.
(D) A rare virus infected a group of people a decade ago;
therefore, it will not reinfect the same population
now.
(E) Each team member has failed to live up to people’s
expectations; therefore, the team will not live up to
people’s expectations.
IMO C
Argument is
X is good but since has never won therefore x will never win
Only C fits the description.
Patient Advocacy Association: In a recent ranking of state hospitals, Hospital A received the lowest overall grade based on a patient mortality rate that was nearly twice that of the top-ranked hospital. Mortality rates in five departments (cardiology, oncology, surgery, neurology, and neonatology) were used in the comparison. Clearly, Hospital A provides the lowest quality of care state-wide in these departments.
One reason that the strength of the Patient Advocacy Association’s argument cannot be evaluated is that
(A) The argument makes no mention of deaths of patients seen by multiple departments.
(B) The Patient Advocacy Association has a reputation as a biased third party that does not always objectively weigh the evidence regarding quality of care.
(C) The Patient Advocacy Association has only cited the mortality rate in five departments of the hospitals.
(D) The Patient Advocacy Association has not included data about the underlying health and severity of illness of the individuals served by each of the hospitals.
(E) The Patient Advocacy Association has not included recent trends in mortality rates by hospital.
D for obvious reasons
In 1992 outlaw fishing boats began illegally harvesting lobsters from the territorial waters
of the country of Belukia. Soon after, the annual tonnage of lobster legally harvested in
Belukian waters began declining; in 1996, despite there being no reduction in the level of
legal lobster fishing activity, the local catch was 9,000 tons below pre-1992 levels. It is
therefore highly likely that the outlaw fishing boats harvested about 9,000 tons of lobster
illegally that year.
Which of the following is an assumption on which the argument depends?
A. The illegal lobster harvesting was not so extensive that the population of
catchable lobsters in Belukia’s territorial waters had sharply declined by 1996.
B. The average annual lobster catch, in tons, of an outlaw fishing boat has increased
steadily since 1992.
C. Outlaw fishing boats do not, as a group, harvest more lobsters than do licensed
lobster-fishing boats.
D. The annual legal lobster harvest in Belukia in 1996 was not significantly less than
9,000 tons.
E. A significant proportion of Belukia’s operators of licensed lobster-fishing boats
went out of business between 1992 and 1996.
Clueless!Any thoughts
A
gtvisa2002 wrote:
The argument says all the 9,000 tons were caught by illegal harvesting.
However, illegal harvesting started in 1992 and both illegal and legal harvesting co-exist from 1992 to 1996.
So obviously the number of tons harvested is steadily increasing.
What if, the steady increase, reduced the mature lobsters which are ready to offspring in the next year…..
So constantly the source is depleted. This provides an alternative explanation to the decrease instead of assuming the number of lobsters harvested is the same throughout the period 1992-96.
Can you be little more elaborate….
Ok let me try.
Say in 1992 there were 100,000 tons of lobsters. Illegal harvesting:10,000 tons Legal:20,000 tons so we are left with 70,000 tons.
Assuming the harvesting rate remains the same, we started 1992 with 100,000, 1993 with 70,000 : we will have only 40,000 for 1994, 10,000 for 1995 and for 1996 we will not have any lobsters available.
The argument assumes that every year this 100,000 tons remains the same in the starting of every year so the lost 9,000 tons were caught by illegal harvesting team.
Option A says that the population remained the same, confirming the assumption.
Governments have only one response to public criticism of socially necessary services: regulation of the activity of providing those services. But governments inevitably make the activity more expensive by regulating it, and that is particularly troublesome in these times of strained financial resources. However, since public criticism of child-care services has undermined all confidence in such services, and since such services are socially necessary, the government is certain to respond.
Which one of the following statements can be inferred from the passage?
(A) The quality of child care will improve.
(B) The cost of providing child-care services will increase.
(C) The government will use funding to foster advances in child care.
(D) If public criticism of policy is strongly voiced, the government is certain to respond.
(E) If child-care services are not regulated, the cost of providing child care will not increase.
OA is B
- With Proposition 13, if you bought your house 11 years ago for $75,000, your property tax would be approximately $914 a year (1 percent of $75,000 increased by 2 percent each year for 11 years); and if your neighbor bought an identical house next door to you for $200,000 this year, his tax would be $2,000 (1 percent of $200,000). Without Proposition 13, both you and your neighbor would pay $6,000 a year in property taxes (3 percent of $200,000).
Which of the following is the conclusion for which the author most likely is arguing in the passage above?
(A) Proposition 13 is unconstitutional because it imposes an unequal tax on properties of equal value.
(B) If Proposition 13 is repealed, every homeowner is likely to experience a substantial increase in property taxes.
(C) By preventing inflation from driving up property values, Proposition 13 has saved homeowners thousands of dollars in property taxes.
(D) If Proposition 13 is not repealed, identical properties will continue to be taxed at different rates.
(E) Proposition 13 has benefited some homeowners more than others.
OA after some explanations
B
With proposition 13, they pay less property tax; while one would be $914, $2000 other. If it is repealed, then each has to pay $6000.
If you elaborate the question:
Without proposition 13 , the value of the property will be revalued. (Each house will be $200.000, moreover the assumption is based on the identical houses.) With proposition 13, you pay a fixed property tax(%1 for years); without proposition 13, you pay a current rate (%3).
B
With proposition 13, they pay less property tax; while one would be $914, $2000 other. If it is repealed, then each has to pay $6000.
If you elaborate the question:
Without proposition 13 , the value of the property will be revalued. (Each house will be $200.000, moreover the assumption is based on the identical houses.) With proposition 13, you pay a fixed property tax(%1 for years); without proposition 13, you pay a current rate (%3).
E
In a monogamous culture, 100% of the adults are married. The average number of children per family is five and over-population is a threat. Programs to encourage birth-control have been ineffective. It has been suggested that this failure is due to these programs ignoring a tradition that values male children very highly, so that every parent wants to have at least one son. It is proposed that couples be encouraged to use birth-control measures after the birth of their first son.
If this proposal is widely accepted in the culture, we may expect that:
(A) the rate of population increase will be slowed, and future generations will contain a disproportionately high number of females.
(B) the rate of population increase will be slowed, and the gender balance in future generations will remain as it is at present.
(C) the rate of population growth will remain the same, and future generations will contain a disproportionately high number of females.
(D) there will be no significant effect either on population growth or on gender balance.
(E) the population will decline precipitously, because approximately half of all families will have only a single child
A
IMO A
Since people will start using birth control after one son therefore population growth will decline. This rules out C and D.
Since most families will have a maximum of 1son whereas other families have two, three or more daughters.
Not enough information is provided to chose E. even if 50% of the population has one son, we don’t know what percentage would have one, two, three and more daughters with at max one or no sons. Don’t expect GMAT questions to expect that without this infomation we can find the solution. Therefore ruled out E
Between A and B, I ruled out B because all families will have at max one son whereas there is no limit to girls.even if 50% of the population will have max one son then the rest of 50% will have one or more than one daughters. Hence there will be a disproportionate number of females. This rules out B
Answer is A
At one time, European and Japanese companies tried to imitate their American rivals. Today, American appliance manufacturers import European scientists to lead their research staffs; American automakers design cars that mimic the styling of German, Italian, and French imports; and American electronics firms boast in their advertising of Japanese-style devotion to quality and reliability. In the world of high technology, America has lost the battle for international prestige.
Each of the following statements, if true, would help to support the claim above EXCEPT:
(A) An American camera company claims in its promotional literature to produce cameras as fine as the best Swiss imports.
(B) An American maker of stereo components designs its products to resemble those of a popular Japanese firm.
(C) An American manufacturer of video games uses a brand name chosen because it sounds like a Japanese word.
(D) An American maker of televisions studies German-made televisions in order to adopt German manufacturing techniques.
(E) An American maker of frozen foods advertises its dinners as Real European-style entrees prepared by fine French and Italian chefs.
IMO E
We are talking about technology. Not food stuff. Option E is the odd one out.
In the effort to fire a Civil Service employee, his or her manager may have to spend up to $100,000 of tax money. Since Civil Service employees know how hard it is to fire them, they tend to loaf. This explains in large part why the government is so inefficient.
It can be properly inferred on the basis of the statements above that the author believes which of the following?
I. Too much job security can have a negative influence on workers.
II. More government workers should be fired.
III. Most government workers are Civil Service employees.
(A) I only
(B) I and III only
(C) II only
(D) I, II, and III
(E) III only
Tricky problem!, but I think the answer is A.
I. Too much job security can have a negative influence on works. This is a very close and logical flow from the statement, which makes two distinct points–firstly, that Civil Service employees know how hard it is to fire them, and secondly, that because they know how hard it is to be fired they tend to loaf. Therefore, their job security causes them to underperform.
II. More government workers should be fired. This can not be confirmed in the statement, because the final sentence is unclear about which statement explains why the government is so inefficient. Is it because it costs up to $100,000 just to fire someone OR is it because people loaf due to job security? It’s too ambiguous to tell.
III. Most government workers are Civil Service employees. There is no reference to number at all in the statement.
Therefore, A.
Tooten County is populated by a significantly lower percentage of young families than the typical county in the nation. Yet, per capita retail sales of children’s bicycles are above the national average.
Which of the following, if true, best reconciles the seeming paradox described above?
A. The two leading retailers of children’s bicycles in Tooten County spend a remarkable 50% of their marketing budget on attracting residents of neighboring counties.
B. A local manufacturer of children’s bicycles has just announced that national sales are up 20% over last year.
C. Retail outlets in Tooten County import the majority of their children’s bicycles from Canada.
D. Elementary schools in Tooten County continue to promote the values of exercise to its students.
E. Residents of Tooten County rent children’s bicycles as about the same rate as residents of other counties in the nation.
IMO - D OA - not known
A
Tooten County is populated by a significantly lower percentage of young families than the typical county in the nation. Yet, per capita retail sales of children’s bicycles are above the national average.
…lower % of young families…
Higher sales of children bicycle….
Both statements contradict as there are fewer younger people in the county. But sales for children bicycle is high.
Which of the following, if true, best reconciles the seeming paradox described above?
A. The two leading retailers of children’s bicycles in Tooten County spend a remarkable 50% of their marketing budget on attracting residents of neighboring counties.
Reconciles the anomaly as younger people from other counties visit and purchase children bicycles. Since there are fewer children in tootie county therefore customers are from neighboring areas.
B. A local manufacturer of children’s bicycles has just announced that national sales are up 20% over last year.
No context. Does not link or explain the difference between the two statements in blue. In fact this just rephrased the 2nd sentence. The question still remains unanswered that if there are fewer younger people in the county then who is buying the cycles.
C. Retail outlets in Tooten County import the majority of their children’s bicycles from Canada.
Again does not answer who is buying the products.
D. Elementary schools in Tooten County continue to promote the values of exercise to its students.
Even if elementary schools promote exercise one young adult will only buy one bicycle. The question remains if there are fewer younger people who are going to elementary school, does it justify a higher per capita in sale of bicycles. Even if all the young adults buy the bicycles even then sales of children bicycles should not be higher considering there are fewer children there.
E. Residents of Tooten County rent children’s bicycles as about the same rate as residents of other counties in the nation.
No context. Does not answer who is buying the bicycles.
IMO - D OA - not known
Ace Automotive Group, a publicly held manufacturer of cars and light trucks, plans to move the production of engines for its automobiles to Mexico from plants in Indiana. However, Ace Automotive will still assemble its cars and trucks in the United States. The president of Ace Automotive announced to Wall Street analysts that this move will save the company $20 million per year even after accounting for the increased transportation costs needed to get the Mexican produced engines to Ace’s assembly plants versus current engine transportation costs.
The statements above, if true, best support which of the following assertions?
A. Ace Automotive Group’s stock price will rise.
B. Ace Automotive will have to deal with pickets and protests when it closes its engine production facilities in Indiana.
C. Ace Automotive would save more than $20 million per year from this move if engine transportation charges from Mexico equaled the old transportation charges from Indiana.
D. Ace Automotive Group’s assembly plants are, on average, closer to the old Indiana engine plants than they are to the new Mexican engine plants.
E. Ace plans to spend less than $20 million on increased transportation costs.
IMO - D OA - not known.
I am confused in D because increased transportation cost does not necessarily mean increased distance.
But D looks best by POE.
C
Ace Automotive Group, a publicly held manufacturer of cars and light trucks, plans to move the production of engines for its automobiles to Mexico from plants in Indiana. However, Ace Automotive will still assemble its cars and trucks in the United States. The president of Ace Automotive announced to Wall Street analysts that this move will save the company $20 million per year even after accounting for the increased transportation costs needed to get the Mexican produced engines to Ace’s assembly plants versus current engine transportation costs.
The statements above, if true, best support which of the following assertions?
A. Ace Automotive Group’s stock price will rise.
We donot know the company,s financial position or public sentiment therefore cannot comment on the Impact on the stok price. Also it is possible that the company may need to incur additional costs for tranortation tfrom Mexico to assembly p,ant.
B. Ace Automotive will have to deal with pickets and protests when it closes its engine production facilities in Indiana.
We donot know how satisfied or unsatisfied the companys workers are. Or what will be the impact on their morale.
C. Ace Automotive would save more than $20 million per year from this move if engine transportation charges from Mexico equaled the old transportation charges from Indiana.
written that the company expects to save $20 million per year even after accounting for INCREASED tranortation costs…….. If the costs are the same I.e there is no increase then the company would be able to save more than $20 million. Hence C should be the answer.
D. Ace Automotive Group’s assembly plants are, on average, closer to the old Indiana engine plants than they are to the new Mexican engine plants.
We donot know whether the plants are closer or not just that transportation cost will rise. The terrain could be difficult and Mexico plant can be close. So we really donot know
E. Ace plans to spend less than $20 million on increased transportation costs.
No context. As $ 20million is what would be saved not the total transportation. Cost.
Answer should be C
I am confused in D because increased transportation cost does not necessarily mean increased distance.
Elapsed Time: 00:00startlapstop
Only a reduction of 10 percent in the number of scheduled flights using Greentown’s airport will allow the delays that are so common there to be avoided. Hevelia airstrip, 40 miles away, would, if upgraded and expanded, be an attractive alternative for fully 20 percent of the passengers using Greentown airport. Nevertheless, experts reject the claim that turning Hevelia into a full-service airport would end the chronic delays at Greentown.
Which of the following, if true, most helps to justify the experts’ position?
(A) Turning Hevelia into a full-service airport would require not only substantial construction at the airport itself, but also the construction of new access highways.
(B) A second largely undeveloped airstrip close to Greentown airport would be a more attractive alternative than Hevelia for many passengers who now use Greentown.
(C) Hevelia airstrip lies in a relatively undeveloped area but would, if it became a full-service airport, be a magnet for commerc ial and residential development.
(D) If an airplane has to wait to land, the extra jet fuel required adds significantly to the airline’s costs.
(E) Several airlines use Greentown as a regional hub, so that most flights landing at Greentown have many passengers who then take different flights to reach their final destinations.
I arrived at the correct answer through the Process of elimination , but I still couldn’t figure out why E is correct
E
IMO E
Only one that shows that the new airport Would not prevent delays because other airlines use green… Airport for connecting flights.
Environmentalist: The commissioner of the Fish and Game Authority would have the public believe that increases in the number of marine fish caught demonstrate that this resource is no longer endangered. This is a specious argument, as unsound as it would be to assert that the ever-increasing rate at which rain forests are being cut down demonstrates a lack of danger to that resource. The real cause of the increased fish-catch is a greater efficiency in using technologies that deplete resources.
The environmentalist’s statements, if true, best support which of the following as a conclusion?
(A) The use of technology is the reason for the increasing encroachment of people on nature.
(B) It is possible to determine how many fish are in the sea in some way other than by catching fish.
(C) The proportion of marine fish that are caught is as high as the proportion of rain forest trees that are cut
down each year.
(D) Modern technologies waste resources by catching inedible fish.
(E) Marine fish continue to be an endangered resource.
What I understand from the argument is:
The commissioner wants the public to believe that marine fish is not endangered anymore, whereas , it is the reverse case.
I think even A support’s the conclusion well enough by saying that technology is the root cause for encroachment ?
IMO E
Commissioner says fish are not endangered.
Environmentalist argues and gives a conclusion that fish are still endangered.
The environmentalist is not talking against technology but is givi g a counter argument to the commissioners argument.
Flag
In the country of Veltria, the past two years’ broad economic recession has included a business downturn in the clothing trade, where sales are down by about 7 percent as compared to two years ago. Clothing wholesalers have found, however, that the proportion of credit extended to retailers that was paid off on time fell sharply in the first year of the recession but returned to its prerecession level in the second year.Which of the following, if true, most helps to explain the change between the first and the second year of the recession in the proportion of credit not paid off
on time?
(A) The total amount of credit extended to retailers by clothing wholesalers increased between the
first year of the recession and the second year.
(B) Between the first and second years of the recession, clothing retailers in Veltria saw many
of their costs, rent and utilities in particular, increase.
(C) Of the considerable number of clothing retailers in Veltria who were having financial difficulties
before the start of the recession, virtually all were forced to go out of business during its
first year.
(D) Clothing retailers in Veltria attempted to stimulate sales in the second year of the
recession by discounting merchandise.
(E) Relatively recession-proof segments of the clothing trade, such as work clothes, did not
suffer any decrease in sales during the first year of the recession.
IMO C
Debtors did not pay on time as weak businesses went out of business.
A major network news organization experienced a drop in viewership in the week following the airing of a controversial report on the economy. The network also received a very large number of complaints regarding the report. The network, however, maintains that negative reactions to the report had nothing to do with its loss of viewers.
Which of the following, if true, most strongly supports the network’s position?
A.The other major network news organizations reported similar reductions in viewership during the same week.
B.The viewers who registered complaints with the network were regular viewers of the news organization’s programs.
C.Major network news organizations publicly attribute drops in viewership to their own reports only when they receive complaints about those reports.
D.This was not the first time that this network news organization has aired a controversial report on the economy that has inspired viewers to complain to the network.
E. Most network news viewers rely on network news broadcasts as their primary source of information regarding the economy.
Can anyone please explain why E is wrong? I assumed that E is correct because most network news viewers rely on the network news for primary source of information so the viewership will not drop as the viewers will have no more alternatives to get the information regarding the economy.
Please correct me if my understanding is wrong.
IMO a
Only reason which justifies that drop in viewership was not because of the report.
IMO a
Only reason which justifies that drop in viewership was not because of the report.
I believe the answer is D.
The argument states that the programming bug will only be in the temporary files and that they are purged after 90 days meaning that no bug associated with the program will affect the computer after 90 days.
Weaken: If after 90 days that program bug is still freezing the computer, then clearly the file has not only affected the temporary file but other files that are purged at lesser frequency thus weakens the argument.
Editorial: The roof of Northtown Council’s equipment-storage building collapsed under
the weight of last week’s heavy snowfall. The building was constructed recently and met
local building-safety codes in every particular, except that the nails used for attaching
roof supports to the building’s columns were of a smaller size than the codes specify for
this purpose. Clearly, this collapse exemplifies how even a single, apparently
insignificant, departure from safety standards can have severe consequences.
Which of the following, if true, most strongly supports the editorial’s argument?
A. The only other buildings whose roofs collapsed from the weight of the snowfall
were older buildings constructed according to less exacting standards than those
in the safety codes.
B. Because of the particular location of the equipment-storage building, the weight
of snow on its roof was greater than the maximum weight allowed for in the
safety codes.
C. Because the equipment-storage building was not intended for human occupation,
some safety-code provisions that would have applied to an office building did not
apply to it.
D. The columns of the building were no stronger than the building-safety codes
required for such a building.
E. Because the equipment-storage building was where the council kept snowremoval
equipment, the building was almost completely empty when the roof
collapsed.
IMHO A,
I would agree with D if it say no weaker than the code requires,
A birth is more likely to be difficult when the mother is over the age of 40 than when she is younger. Regardless
of the mother’s age, a person whose birth was difficult is more likely to be ambidextrous than is a person whose
birth was not difficult. Since other causes of ambidexterity are not related to the mother’s age, there must be
more ambidextrous people who were born to women over 40 than there are ambidextrous people who were born
to younger women. The argument is most vulnerable to which one of the following criticisms?
A. It assumes what it sets out to establish.
B. It overlooks the possibility that fewer children are born to women over 40 than to women under 40.
C. It fails to specify what percentage of people in the population as a whole are ambidextrous.
D. It does not state how old a child must be before its handedness can be determined.
E. It neglects to explain how difficulties during birth can result in a child’s ambidexterity.
B not A
The argument is that more ambidextrous people are born to women over 40. In no way is this assumed in the passage. You are looking to refute the number and this conclusion should give a relative number not an absolute.
Shareholders and investment analysts have criticized the Vista Products Company for having slow and outdated computer systems in its factories when these systems are compared to Vista’s competitor’s systems. However, a Vista representative has claimed that such criticism is unfair. The representative noted that Vista spent more money last year per factory upgrading its computer systems than any of its competitors did.
Which of the following, if true, most seriously discredits the reasoning in the company representative’s argument?
A. Overall spending per factory on computer systems is lower in Vista Product’s industry than in other industries.
B. Vista Products is not as profitable as some of its competitors
C. Companies with fast, modern computer systems would have no need to spend large amounts of money to upgrade these systems.
D. No investment analyst has visited every single Vista Products factory.
E. When assessing the overall quality of a company’s operations, production line speed is more critical than the quality of the company’s computer systems.
C
IMO C
Only answer that weekends the argument.
The number of aircraft collisions on the ground is increasing because of the substantial increase in the number of flights operated by the airlines. Many of the fatalities that occur in such collisions are caused not by the collision itself, but by an inherent flaw in the cabin design of most aircraft, in which seats, by restricting access to emergency exits, impede escape. Therefore, to reduce the total number of fatalities that result annually from such collisions, the airlines should be required to remove all seats that restrict access to emergency exits.
Which one of the following proposals, if implemented together with the proposal made in the passage, would improve the prospects for achieving the stated objective of reducing fatalities?
(A) The airlines should be required, when buying new planes, to buy only planes with unrestricted access to emergency exits.
(B) The airlines should not be permitted to increase further the number of flights in order to offset the decrease in the number of seats on each aircraft.
(C) Airport authorities should be required to streamline their passenger check-in procedures to accommodate the increased number of passengers served by the airlines.
(D) Airport authorities should be required to refine security precautions by making them less conspicuous without making them less effective.
(E) The airlines should not be allowed to increase the ticket price for each passenger to offset the decrease in the number of seats on each aircraft.
hmmm….my pick B
Quote:
Which one of the following proposals, if implemented together with the proposal made in the passage, would improve the prospects for achieving the stated objective of reducing fatalities?
two reasons for number of aircraft collisions- more no. of flights and blocked exits The latter one i.e.remove restriction to exits is already mentioned in the conclusion-Therefore, to reduce the total number of fatalities that result annually from such collisions, the airlines should be required to remove all seats that restrict access to emergency exits
the other is- curtailing the increase of number of flights is to be addresed….as the airlines could try and increase the no.of flights to make up for the lost profits incurred by the removal of seats thereby increasing the no. of flights and risking more collisions ……hence B fits the bill
q) In the country of Veltria, the past two years’ broad economic recession has included a business downturn in the clothing trade, where sales are down by about 7 percent as compared to two years ago. Clothing wholesalers have found, however, that the proportion of credit extended to retailers that was paid off on time fell sharply in the first year of the recession but returned to its pre recession level in the second year. Which of the following, if true, most helps to explain the change between the first and the second year of the recession in the proportion of credit not paid off on time?
(A) The total amount of credit extended to retailers by clothing wholesalers increased between the first year of the recession and the second year.
(B) Between the first and second years of the recession, clothing retailers in Veltria saw many of their costs, rent and utilities in particular, increase.
(C) Of the considerable number of clothing retailers in Veltria who were having financial difficulties before the start of the recession, virtually all were forced to go out of business during its first year.
(D) Clothing retailers in Veltria attempted to stimulate sales in the second year of the recession by discounting merchandise.
(E) Relatively recession-proof segments of the clothing trade, such as work clothes, did not suffer any decrease in sales during the first year of the recession.
IMO C
.kumar wrote:
q) In the country of Veltria, the past two years’ broad economic recession has included a business downturn in the clothing trade, where sales are down by about 7 percent as compared to two years ago. Clothing wholesalers have found, however, that the proportion of credit extended to retailers that was paid off on time fell sharply in the first year of the recession but returned to its pre recession level in the second year. Which of the following, if true, most helps to explain the change between the first and the second year of the recession in the proportion of credit not paid off on time?
(A) The total amount of credit extended to retailers by clothing wholesalers increased between the first year of the recession and the second year.
(B) Between the first and second years of the recession, clothing retailers in Veltria saw many of their costs, rent and utilities in particular, increase.
(C) Of the considerable number of clothing retailers in Veltria who were having financial difficulties before the start of the recession, virtually all were forced to go out of business during its first year.
(D) Clothing retailers in Veltria attempted to stimulate sales in the second year of the recession by discounting merchandise.
(E) Relatively recession-proof segments of the clothing trade, such as work clothes, did not suffer any decrease in sales during the first year of the recession.
OA is C but IMO E, Please kindly explain
Credit paid back fell in first year and in the second year it came back to normal level. Even though recession affected two years, how did it happen? We need explain this situation:
E Says that some clothing segments were doing well in the first year. It doesn’t tell anything about the second year. We need an explanation that answers why the credit paying was back to normal in the second year, after having fallen in the first. C answers that question successfully.
C says that those (retailers having financial difficulties) who couldn’t payback in the first year, all went out of business. So, by the second year only those who didn’t have financial difficulties (successful) remained in the market. So, if only successful retailers remained in the market by second year, they all paid the credit back, restoring it to the normal level.
Hope it helps.
_________________
- VK
Learn. Recognize. Apply
Though sucking zinc lozenges has been promoted as a treatment for the common cold, research has revealed no consistent effect. Recently, however, a zinc gel applied nasally has been shown to greatly reduce the duration of colds. Since the gel contains zinc in the same form and concentration as the lozenges, the greater effectiveness of the gel must be due to the fact that cold viruses tend to concentrate in the nose, not the mouth.
In order to evaluate the argument, it would be most helpful to determine which of the following?
A. Whether zinc is effective only against colds, or also has an effect on other virally caused diseases
B. Whether there are remedies that do not contain zinc but that, when taken orally, can reduce the duration of colds
C. Whether people who frequently catch colds have a zinc deficiency
D. Whether either the zinc gel or the lozenges contain ingredients that have an impact on the activity of the zinc
E. Whether the zinc gel has an effect on the severity of cold symptoms, as well as on their duration
rx_11 wrote:
Though sucking zinc lozenges has been promoted as a treatment for the common cold, research has revealed no consistent effect. Recently, however, a zinc gel applied nasally has been shown to greatly reduce the duration of colds. Since the gel contains zinc in the same form and concentration as the lozenges, the greater effectiveness of the gel must be due to the fact that cold viruses tend to concentrate in the nose, not the mouth.
In order to evaluate the argument, it would be most helpful to determine which of the following?
The Red Bold part is the argument to be evaluated . This is a “relevant information” type question which is a hybrid of both Strengthen and Weaken Question. In the correct choice, one answer to the question will strengthen the argument and the opposite answer will weaken the argument.
Our objective here is to figure out why the gel is more effective than the lozenge.
A. Whether zinc is effective only against colds, or also has an effect on other virally caused diseases
B. Whether there are remedies that do not contain zinc but that, when taken orally, can reduce the duration of colds
Both lozenge and gel contains zinc, so no point in talking about remedies that do not contain zinc
C. Whether people who frequently catch colds have a zinc deficiency
D. Whether either the zinc gel or the lozenges contain ingredients that have an impact on the activity of the zinc
If the gel and lozenge DO contain ingredients that impact the effectiveness of the zinc, the author’s argument is weakened, since those ingredients could be responsible for the difference in effectiveness.
E. Whether the zinc gel has an effect on the severity of cold symptoms, as well as on their duration
Scientist: My research indicates that children who engage in impulsive behavior similar to adult thrill-seeking behavior are twice as likely as other children to have a gene variant that increases sensitivity to dopamine. From this, I conclude that there is a causal relationship between this gene variant and an inclination toward thrill-seeking behavior.
Which one of the following, if true, most calls into question the scientist’s argument?
A) Many impulsive adults are not unusually sensitive to dopamine.
B) It is not possible to reliably distinguish impulsive behavior from other behavior.
C) Children are often described by adults as engaging in thrill-seeking behvaior simply because they act impulsively.
D) Many people exhibit behavioral tendencies as adults that they did not exhibit as children.
E) The gene variant studied by the scientist is correlated with other types of behavior in addition to thrill-seeking behavior.
B
Let me frame my question in direct reference to @amit.trivedi’s reply.
First off, (B) is a resounding, powerful rebuttal of the scientist’s claim (remember we want an answer that most calls into question…
The scientist is making a claim about impulsive behavior in children. However, if impulsive behavior in children is indistinguishable from other behavior, then the whole argument falls apart.
For (E) we have a tempting answer, especially because the word ‘correlation’ suggest that a causal connection is not definitive. But let’s say the gene variant is also linked to overeating (i.e., another type of behavior). How does this weaken claim that the gene variant causes impulsive behavior? In fact, the gene variant could be associated with 50 other types of behavior. That fact doesn’t diminish the causal connection—–> Gene variant causes impulsive behavior.
The price the government pays for standard weapons purchased from military contractors is determined by a pricing method called “historical costing’’. historical costing allows contractors to protect their profits by adding a percentage increase based on the current rate of inflation, to the previous year’s contractual price.
Which of the following statements, if true, is the best basis for a criticism of historical costing as an economically sound pricing method for military
contracts?
(A) The government might continue to pay for
past inefficient use of funds.
(B) The rate of inflation has varied considerably
over the past twenty years.
(C) The contractual price will be greatly affected
by the cost of materials used for the products.
(D) Many taxpayers question the amount of
money the government spends on military
contracts.
E) The pricing method based on historical costing might not encourage the development of innovative weapons.
(A) The government might continue to pay for
past inefficient use of funds. ( correct because historical accounting is based on the previous year’s contractual price and the inflation. Inflation is market driven and hence fluctuates year over year - provides the contractors with a logical playing ground. let’s look at the “contractual price “ of the previous year. Say if in first year of purchase , the government used the funds inefficiently and hence paid more price for the goods. The price it pays for a certain set of goos gets fixed and the second year the prices are based on the first year’s contractual price + inflation . But the contractual price was skewed because the government probably paid more for lesser goods or whatever be the reason for inefficiency
(B) The rate of inflation has varied considerably
over the past twenty years. (this will strengthen the use of historical costing)
(C) The contractual price will be greatly affected
by the cost of materials used for the products. (wrong because everytime the contractual price is based on the previous year’s contractual price + current inflation)
(D) Many taxpayers question the amount of
money the government spends on military
contracts. (out of scope)
E) The pricing method based on historical costing might not encourage the development of innovative weapons. (out of scope)
Historian: Newton developed mathematical concepts and techniques that are fundamental to modern calculus. Leibniz developed closely analogous concepts and techniques. It has traditionally been thought that these discoveries were independent. Researchers have, however, recently discovered notes of Leibniz’ that discuss one of Newton’s books on mathematics. Several scholars have argued that since the book includes a presentation of Newton’s calculus concepts and techniques, and since the notes were written before Leibniz’ own development of calculus concepts and techniques, it is virtually certain that the traditional view is false. A more cautious conclusion than this is called for, however. Leibniz’ notes are limited to early sections of Newton’s book, sections that precede the ones in which Newton’s calculus concepts and techniques are presented.
In the historian’s reasoning, the two boldfaced portions play which of the following roles?
A. The first provides evidence in support of the overall position that the historian defends; the second is evidence that has been used to support an opposing position.
B. The first provides evidence in support of the overall position that the historian defends; the second is that position.
C. The first provides evidence in support of an intermediate conclusion that is drawn to provide support for the overall position that the historian defends; the second provides evidence against that intermediate conclusion.
D. The first is evidence that has been used to support a conclusion that the historian criticizes; the second is evidence offered in support of the historian’s own position.
E. The first is evidence that has been used to support a conclusion that the historian criticizes; the second is further information that substantiates that evidence.
Answer: D (not confirmed)
Which of the following best completes the passage below?
A primary factor in perpetuating the low salaries of women workers has been their segregation in the so-called pink-collar occupations, such as nursing, teaching, library science, and secretarial work. Partly because these jobs have traditionally been held by women, their salary levels have been depressed, and, despite increased attempts to unionize these workers in recent years, their pay continues to lag. Moreover, although a large percentage of women than ever before are now entering and remaining in the jobs market, most continue to gravitate toward the pink-collar fields, despite the lower salaries. It seems clear, therefore, that if the average salaries of women workers are to approach those of men, ______
(A) labor unions must redouble their efforts to improve the lot of working women
(B) society’s perception of pink-collar jobs as less important and less demanding than other jobs must be changed
(C) more men must be encouraged to enter fields traditionally occupied by women
(D) the number of jobs in the pink-collar fields relative to the size of the work force as a whole must be markedly increased
(E) more women must enter occupations other than those traditionally reserved for them
Answer: E
E
Every political philosopher of the early twentieth century who was either a socialist or a communist was influenced by Rosa Luxemburg. No one who was influenced by Rosa Luxemburg advocated a totalitarian state.
If the statements above are true, which one of the following must on the basis of them also is true?
(A) No early-twentieth-century socialist political philosopher advocated a totalitarian state.
(B) Every early-twentieth-century political philosopher who did not advocate a totalitarian state was influenced by Rosa Luxemburg.
(C) Rosa Luxemburg was the only person to influence every early-twentieth-century political philosopher who was either socialist or communist.
(D) Every early-twentieth-century political philosopher who was influenced by Rosa Luxemburg and was not a socialist was a communist.
(E) Every early-twentieth-century political philosopher who did not advocate a totalitarian state was either socialist or communist.
Answer: A
A
Some environmentalists question the prudence of exploiting features of the environment, arguing that there are no economic benefits to be gained from forests, mountains, or wetlands that no longer exist. Many environmentalists claim that because nature has intrinsic value it would be wrong to destroy such features of the environment, even if the economic costs of doing so were outweighed by the economic costs of not doing so. Which one of the following can be logically inferred from the passage? (A) It is economically imprudent to exploit features of the environment. (B) Some environmentalists appeal to a noneconomic justification in questioning the defensibility of exploiting features of the environment. (C) Most environmentalists appeal to economic reasons in questioning the defensibility of exploiting features of the environment. (D) Many environmentalists provide only a noneconomic justification in questioning the defensibility of exploiting features of the environment. (E) Even if there is no economic reason for protecting the environment, there is a sound noneconomic justification for doing so.
B
Explanation: the justification advanced by “many environmentalists” in the last sentence is clearly non-economic, since it runs directly counter to economic principles (quote: … even if the economic costs of doing so were outweighed by the economic costs of not doing so).
i’ll explain why C and D are wrong.
(c) While there is an economic justification in the passage - specifically, the justification mentioned in the first sentence - this justification is limited to SOME environmentalists. this choice is wrong, then, since it claims “most” and is thus overreaching.
(d) This choice is wrong because of the “only”.
The passage claims that certain environmentalists have advanced a non-economic justification, but never says that this is the ONLY justification advanced by those environmentalists.
In fact, note that, REGARDLESS OF THE QUESTION, this CANNOT be the correct answer IF (b) is also an available option. (reason: if (d) is true, then (b) MUST also be true, since it’s a weaker claim about the same thing. therefore, if (d) is true, then (b) is a fortiori also true. since you can’t have two correct answers, (d) can’t possibly be correct.)
Economist: In the interaction between producers and consumers, the only obligation that all parties have to act in the best interests of their own side. And distribution of information about product defects is in the best interests of the consumer, So consumers are always obligated to report product defects they discover, while producers are never obligated to reveal them. Which one of the following is an assumption required by the economist’s argument? A. It is never in the best interests of producers for a producer to reveal a product defect. B. No one expects producers to act in a manner counter to their own best interests. C. Any product defect is likely to be discovered by consumer D. A product defect is more likely to be discovered by a consumer than by a producer E. The best interests of consumers never coincide with the best interests of producers
Answer: A
Explanation:
Argument:In the interaction between producers and consumers, the only obligation that all parties have to act in the best interests of their own side.
Conclusion : So consumers are always obligated to report product defects they discover, while producers are never obligated to reveal them.
Assumption : It is never in the best interests of producers for a producer to reveal a product defect.
A
The workers at Bell Manufacturing will shortly go on strike unless the management increases their wages. As Bell’s president is well aware, however, in order to increase the worker’s wages, Bell would have to sell off some of its subsidiaries. So, some of Bell’s subsidiaries will be sold.
The conclusion above is properly drawn if which one of the following is assumed?
(A) Bell Manufacturing will begin to suffer increased losses.
(B) Bell’s management will refuse to increase its worker’s wages.
(C) The workers at Bell Manufacturing will not be going on strike.
(D) Bell’s president has the authority to offer the workers their desired wage increase.
(E) Bell’s workers will not accept a package of improved benefits in place of their desired wage increase.
Answer: C
C
Advertisement: Anyone who exercises knows from firsthand experience that exercise leads to better performance of such physical organs as the heart and lungs, as well as to improvement in muscle tone. And since your brain is a physical organ, your actions can improve its performance, too. Act now. Subscribe to Stimulus: read the magazine that exercises your brain.
The Advertisement employs which one of the following argumentative strategies?
(A) It cites experimental evidence that subscribing to the product being advertised has desirable consequences.
(B) It ridicules people who do not subscribe to Stimulus by suggesting that they do not believe that exercise will improve brain capacity.
(C) It explains the process by which the product being advertised brings about the result claimed for its use.
(D) It supports its recommendation by a careful analysis of the concept of exercise.
(E) It implies that brains and muscle are similar in one respect because they are similar in another respect.
Answer: E
E
Explanation: since the ad states that working out your brain will improve your brain much in the same way that working out your physical organs improve their performance
When Alicia Green borrowed a neighbor’s car without permission, the police merely gave her a warning. However, when Peter Foster did the same thing, he was charged with automobile theft. Peter came to the attention of the police because the car he was driving was hit by a speeding taxi. Alicia was stopped because the car she was driving had defective taillights. It is true that the car Peter took got damaged and the car Alicia took did not, but since it was the taxi that caused the damage this difference was not due to any difference in the blameworthiness of their behavior. Therefore, Alicia should also have been charged with automobile theft. If all of the claims offered in support of the conclusion are accurate, each of the following could be true EXCEPT: (A) The interests of justice would have been better served if the police had released Peter Foster with a warning. (B) Alicia Green had never before driven a car belonging to someone else without first securing the owner’s permission. (C) Peter Foster was hit by the taxi while he was running a red light, whereas Alicia Green drove with extra care to avoid drawing the attention of the police to the car she had taken. (D) Alicia Green barely missed hitting a pedestrian when she sped through a red light ten minutes before she was stopped by the police for driving a car that had defective taillights. (E) Peter Foster had been cited for speeding twice in the preceding month, whereas Alicia Green had never been cited for a traffic violation.
Answer: C
Explanation: The part of the argument that’s important is: but since it was the taxi that caused the damage this difference was not due to any difference in the blameworthiness of their behavior.. This part of the argument states that neither driver was at fault or to blame. However, choice C states that Peter ran a red light, which would imply that he was at least partially to blame.
Under the influence of today’s computer-oriented culture, publishing for children has taken on a flashy new look that emphasizes illustrations and graphic design; the resulting lack of substance leads to books that are short-lived items covering mainly trendy subjects. The changes also include more humorous content, simplification of difficult material, and a narrower focus on specific topics.
C
Which one of the following is most strongly supported by the information above?
The inclusion of humorous material and a narrower focus detract from the substance of a children’s book.
The substance of a children’s book is important to its longevity.
Children of the computer generation cannot concentrate on long, unbroken sections of prose.
Children judge books primarily on the basis of graphic design.
The lack of substance of a children’s book is unlikely to be important to its popularity.
Answer: B
Explanation: The prompt says that “the resulting lack of substance leads to books that are short-lived.” If lack of substance leads to lack of longevity, then we can conclude that substance is necessary for longevity.
B
Further evidence of a connection between brain physiology and psychological states has recently been uncovered in the form of a correlation between electroencephalograph patterns and characteristic moods. A study showed that participants who suffered from clinical depression exhibited less left frontal lobe activity than right, while, conversely, characteristically good-natured participants exhibited greater left lobe activity. Thus one’s general disposition is a result of the activity of one’s frontal lobe.
Each of the following, if true, weakens the argument EXCEPT:
Many drugs prescribed to combat clinical depression act by causing increased left lobe activity.
Excessive sleep, a typical consequence of clinical depression, is known to suppress left lobe activity.
Frontal lobe activity is not subject to variation the way general disposition is.
Earlier studies indicated that frontal lobe activity and emotive states are both caused by activity in the brain’s limbic activity.
Social interaction of the kind not engaged in by most clinically depressed people is known to stimulate left lobe activity.
Answer: A Explanation: The author is assuming because low left lobe levels and depression are correlated, the former causes the latter. We can weaken this causal relationship by establishing that 1) Depression causes a loss of left lobe activity (b, e), 2) Some other root cause is responsible for both effects (d), or 3) the findings are simply coincidence and the relationship isn’t reliable (c). A, meanwhile, indicates that depression can be fought by raising left-lobe levels, strengthening the conclusion that lobe levels control disposition.
A
We ought to pay attention only to the intrinsic properties of a work of art. Its other, extrinsic properties are irrelevant to our aesthetic interactions with it. For example, when we look at a painting we should consider only what is directly presented in our experience of it. What is really aesthetically relevant, therefore, is not what a painting symbolizes, but what it directly presents to experience.
The conclusion follows logically if which one of the following is added to the premises?
What an art work symbolizes involves only extrinsic properties of that work.
There are certain properties of our experiences of artworks that can be distinguished as symbolic properties.
Only an artwork’s intrinsic properties are relevant to our aesthetic interactions with it.
It is possible in theory for an artwork to symbolize nothing.
An intrinsic property of an artwork is one that related the work to itself.
Answer: A Explanation: The authors conclusion is that what is important to a painting is what it presents to experience rather than what it symbolizes (we know this by the keyword therefore). His evidence is that extrinsic properties are unimportant, and intrinsic properties are the key. Because he’s shifting from irrelevant extrinsic properties in his evidence to irrelevant symbolism in his conclusion, it MUST be true the symbolism is extrinsic; if any symbolism is intrinsic, then his logic is flawed.
A
The Board of Trustees of the Federici Art Museum has decided to sell some works from its collection in order to raise the funds necessary to refurbish its galleries. Although this may seem like a drastic remedy, the curator had long maintained that among the paintings that the late Ms. Federici collected for the museum were several unsuccessful immature works by Renoir and Cezanne that should be sold because they are of inferior quality and so add nothing to the overall quality of the museum’s collection. Hence, the board’s action will not detract from the museum’s collection. Which one of the following, if true, most weaken the argument? (A) The directors of an art museum can generally raise funds for refurbishing the building in which the museum’s collection is housed by means other than selling parts of its collection. (B) The quality of an art collection is determined not just by the quality of its paintings, but by what development of the artistic talent and ideas of the artists represented. (C) The immature woks by Renoir and Cezanne that were purchased by Ms. Federici were at that time thought by some critics to be unimportant juvenile works. (D) Those people who speculate in art by purchasing artworks merely to sell them at much higher prices welcome inflation in the art market, but curators of art museum regret the inflation in the art market. (E) The best work of a great artist demands much higher prices in the art market than the worst work of that same artist.
Answer: B
In a certain municipality, a judge overturned a suspect’s conviction for possession of an illegal weapon. The suspect had fled upon seeing police and subsequently discarded the illegal weapon after the police gave chase. The judge reasoned as follows: the only cause for the police giving chase was the suspect’s flight; by itself, flight from the police does not create a reasonable suspicion of a criminal act; evidence collected during an illegal chase is inadmissible; therefore, the evidence in this case was inadmissible.
Which one of the following principles, if valid, most helps to justify the judge’s decision that the evidence was inadmissible?
(A) Flight from the police could create a reasonable suspicion of a criminal act as long as other significant factors are involved.
(B) People can legally flee from the police only when those people are not involved in a criminal act at the time.
(C) Police can legally give chase to a person only when the person’s actions have created a reasonable suspicion of a criminal act.
(D) Flight from the police should not itself be considered a criminal act.
(E) In all cases in which a person’s actions have created a reasonable suspicion of a criminal act, police can legally give chase to that person
Answer: C
Explanation: Because the judge concludes the evidence is inadmissible due to the illegal chase. And the chase is illegal (according to the judge) because there were no grounds for reasonable suspicion. Again, by designating it an illegal chase, the judge must believe that police can legally give chase only when reasonable suspicion has been established. In logical terms, this is the contra-positive to the judge’s initial statement.
C
Anthropologist: Violence is an extreme form of aggression, and is distinct from the self-expression sufficient for survival under normal conditions. Human beings in certain situations react to unpleasant stimuli with violence but only because they are conditioned by their culture to react in this manner.
Each of the following can be logically inferred from the anthropologist’s statements EXCEPT:
(A) Not all aggression is violent.
(B) The self-expression required for survival is generally nonaggressive.
(C) Some behaviors are influenced by the cultures in which human beings live.
(D) In normal circumstance, human beings can survive by responding nonviolently.
(E) Violent behavior is a product of one’s cultural environment.
Answer: B
B
Commissioner: I have been incorrectly criticized for having made my decision on the power plant issue prematurely. I based my decision on the report prepared by the neighborhood association and although I have not studied it thoroughly, I am sure that the information it contains is accurate. Moreover, you may recall that when I received input from the neighborhood association on jail relocation, I agreed with its recommendation.
The commissioner’s argument is LEAST vulnerable to which one of the following criticism?
(A) It takes for granted that the association’s information is not distorted by bias.
(B) It draws a conclusion about the recommendations of the association from incomplete recollections.
(C) It takes for granted that the association’s report is the only direct evidence that needed to be considered.
(D) It hastily concludes that the association’s report is accurate without having studied it in detail.
(E) It takes for granted that agreeing with the associations past recommendation helps justify agreeing with its current recommendation
Answer: B
Explanation: the commissioner’s conclusion on the recommendation is based on the trust he has on its accuracy => the commissioner cannot be faulted for incomplete recollections (spotty memory)
All other options are valid criticisms that can be made on the commissioner’s argument.
B
A reason Larson cannot do the assignment is that she has an unavoidable scheduling conflict. On the other
hand, a reason Franks cannot do the assignment is that he does not quite have the assertiveness the task
requires. So, the task must be assigned to Parker, the only supervisor in the shipping department other than
Larson and Franks.
The argument depends on assuming which one of the following?
(A) Larson has the assertiveness the task requires.
(B) The task cannot be assigned to anyone other than a supervisor in the shipping department.
(C) Franks would be assigned the task if Franks had the assertiveness the task requires.
(D) The task cannot be assigned to anyone who has any kind of scheduling conflict.
(E) No one who is not a supervisor in the shipping department has the assertiveness this task requires.
Answer: D
D